summaryrefslogtreecommitdiff
path: root/Master/texmf-dist/doc/latex/bhcexam/test2.tex
blob: 8dac567787b9f244dacd1259179ae6cc20c29feb (plain)
1
2
3
4
5
6
7
8
9
10
11
12
13
14
15
16
17
18
19
20
21
22
23
24
25
26
27
28
29
30
31
32
33
34
35
36
37
38
39
40
41
42
43
44
45
46
47
48
49
50
51
52
53
54
55
56
57
58
59
60
61
62
63
64
65
66
67
68
69
70
71
72
73
74
75
76
77
78
79
80
81
82
83
84
85
86
87
88
89
90
91
92
93
94
95
96
97
98
99
100
101
102
103
104
105
106
107
108
109
110
111
112
113
114
115
116
117
118
119
120
121
122
123
124
125
126
127
128
129
130
131
132
133
134
135
136
137
138
139
140
141
142
143
144
\documentclass[16kpaper]{BHCexam}
\begin{document}

\maketitle
\mininotice

\begin{questions}
\tiankong
\question 已知~$\bm{a}=(k,-9)$、$\bm{b}=(-1,k)$, $\bm{a}$~与~$\bm{b}$~为平行向量,
    则~$k=$\stk{$\pm3$}.

\question 若函数~$f(x)=x^{6m^2-5m-4}\,(m\in\mathbb{Z})$~的图像关于~$y$~轴对称,
    且~$f(2)<f(6)$, 则~$f(x)$~的解析式为\stk{$f(x)=x^{-4}$}.

\question 若~$f(x+1)=x^2\,(x\leq0)$, 则~$f^{-1}(1)=$\stk{0}.

\question 在~$b\text{g}$~糖水中含糖~$a\text{g}$\,($b>a>0$), 若再添加~$m\text{g}$~糖~($m>0$),


\question 已知~$f(x)=1-\textbf{c}_8^1x+\textbf{c}_8^2x^2-\textbf{c}_8^3x^3+\cdots+\textbf{c}_8^8x^8$,
    则~$f\big(\dfrac{1}{2}+\dfrac{\sqrt{3}}{2}\textbf{i}\big)$~的值是\ltk{$-\dfrac{1}{2}-\dfrac{\sqrt{3}}{2}\textbf{i}$}.

\question 自然数~$1,2,3,\ldots,10$~的方差记为~$\sigma^2$,
    其中的偶数~$2,4,6,8,10$~的方差记为~$\sigma_1^2$,
    则~$\sigma^2$~与~$\sigma_1^2$~的大小关系为~$\sigma^2$\stk{$>$}$\sigma_1^2$.

\question 若~$\theta$~为三角形的一个内角, 且~$\sin\theta+\cos\theta=\dfrac{2}{3}$,
    则方程~$x^2\csc\theta-y^2\sec\theta=1$~表示的曲线的焦点坐标是\stk{$\big(\pm\dfrac{\sqrt{6}}{3},0\big)$}.


\question 高为~$h$~的棱锥被平行于棱锥底面的截得棱台侧面积是
    原棱锥的侧面积的~$\dfrac{5}{9}$,
    则截得的棱台的体积与原棱锥的体积之比是\stk{$19:27$}.

\question 以椭圆~$\dfrac{x^2}{169}+\dfrac{y^2}{144}=1$~的右焦点为圆心,
    且与双曲线~$\dfrac{x^2}{9}-\dfrac{y^2}{16}=1$~的渐近线相切的圆方程是\mtk{$(x-5)^2+y^2=16$}.


\question 若~$\sqrt{\,\sin x}$~是有理数且~$x$~不是~$\dfrac{\pi}{6}$~的整数倍,
    则~$x$~可能取的值是\mtk{$\arcsin\dfrac{1}{4}$ 等}.(只要求写出一个)

\question 马路上有编号~1~到~10~的~10~盏路灯, 为节约用电又不影响照明,
    可以关掉其中的~3~盏, 但又不能同时关掉相邻的两盏, 也不能关掉两端的路灯,
    满足条件的关灯方法有\stk{$20$}种.


\question 以椭圆~$\dfrac{x^2}{169}+\dfrac{y^2}{144}=1$~的右焦点为圆心,
    且与双曲线~$\dfrac{x^2}{9}-\dfrac{y^2}{16}=1$~的渐近线相切的圆方程是\mtk{$(x-5)^2+y^2=16$}.

\question 若~$\sqrt{\,\sin x}$~是有理数且~$x$~不是~$\dfrac{\pi}{6}$~的整数倍,
    则~$x$~可能取的值是\mtk{$\arcsin\dfrac{1}{4}$ 等}.(只要求写出一个)

\question 马路上有编号~1~到~10~的~10~盏路灯, 为节约用电又不影响照明,
    可以关掉其中的~3~盏, 但又不能同时关掉相邻的两盏, 也不能关掉两端的路灯,
    满足条件的关灯方法有\stk{$20$}种.


\question 以椭圆~$\dfrac{x^2}{169}+\dfrac{y^2}{144}=1$~的右焦点为圆心,
    且与双曲线~$\dfrac{x^2}{9}-\dfrac{y^2}{16}=1$~的渐近线相切的圆方程是\mtk{$(x-5)^2+y^2=16$}.

\newpage

\xuanze
\question 已知集合~$A=\{x\mid \abs{x-1}<3 \}$,
集合~$B=\{y| y=x^2+2x+1,x\in\mathbb{R}\}$, 则~$A\cap
\complement_U B$~为\stk{C}.
\twoch{$[\,0,4)$}{$(-\infty,-2\,]\cup[4,+\infty)$}{$(-2,0)$}{$(0,4)$}

\question 若~$a$、$b$~是直线, $\alpha$、$\beta$~是平面,
则以下命题中真命题是\stk{D}.\\
\fourch{若~$a$、$b$~异面, $a\subset\alpha$,$b\subset\beta$, 且~$a\perp b$, 则~$\alpha\perp\beta$}{若~$a\parallel b$, $a\subset\alpha$, $b\subset\beta$,则~$\alpha\parallel\beta$}{若~$a\parallel \alpha$,
$b\subset\beta$, 则~$a$、$b$ 异面}{若~$a\perp b$, $a\perp\alpha$,$b\perp\beta$, 则~$\alpha\perp\beta$}

\question 已知集合~$A=\{x\mid \abs{x-1}<3 \}$,
集合~$B=\{y| y=x^2+2x+1,x\in\mathbb{R}\}$, 则~$A\cap
\complement_U B$~为\stk{C}.
\twoch{$[\,0,4)$}{$(-\infty,-2\,]\cup[4,+\infty)$}{$(-2,0)$}{$(0,4)$}

\question 若~$a$、$b$~是直线, $\alpha$、$\beta$~是平面,
则以下命题中真命题是\stk{D}.\\
\fourch{若~$a$、$b$~异面, $a\subset\alpha$,$b\subset\beta$, 且~$a\perp b$, 则~$\alpha\perp\beta$}{若~$a\parallel b$, $a\subset\alpha$, $b\subset\beta$,则~$\alpha\parallel\beta$}{若~$a\parallel \alpha$,
$b\subset\beta$, 则~$a$、$b$ 异面}{若~$a\perp b$, $a\perp\alpha$,$b\perp\beta$, 则~$\alpha\perp\beta$}

\newpage
\jianda
\question 已知复数~$z$ 满足:$\abs{z}-z^*=\dfrac{10}{1-w\textbf{i}}$(其中~$z^*$
是~$z$ 的共轭复数).
\begin{parts}
\part[7] 求复数~$z$;
\part[7] 若复数~$w=\cos\theta+\textbf{i}\sin\theta\,(\theta\in\mathbb{R})$, 求~$\abs{z-2}$ 的取值范围.
\end{parts}

\begin{solution}
\begin{parts}
\part $z=3+4\textbf{i}$
\part $\abs{z-w}\in[4,6]$
\end{parts}
\end{solution}

\question[14] 函数~$f(x)=4\sin\dfrac{\pi}{12}x\cdot\sin
    \left(\dfrac{\pi}{2}+\dfrac{\pi}{12}x\right),x\in[a,a+1]$,
    其中常数~$a\in[0,5]$, 求函数~$f(x)$ 的最大值~$g(a)$.

\begin{solution}
略
\end{solution}

\newpage

\question[16] 函数~$f(x)=4\sin\dfrac{\pi}{12}x\cdot\sin
    \left(\dfrac{\pi}{2}+\dfrac{\pi}{12}x\right),x\in[a,a+1]$,
    其中常数~$a\in[0,5]$, 求函数~$f(x)$ 的最大值~$g(a)$.

\begin{solution}
略
\end{solution}

\newpage
\question 已知复数~$z$ 满足:$\abs{z}-z^*=\dfrac{10}{1-w\textbf{i}}$(其中~$z^*$
是~$z$ 的共轭复数).
\begin{parts}
\part[8] 求复数~$z$;
\part[8] 若复数~$w=\cos\theta+\textbf{i}\sin\theta\,(\theta\in\mathbb{R})$, 求~$\abs{z-2}$ 的取值范围.
\end{parts}

\begin{solution}
\begin{parts}
\part $z=3+4\textbf{i}$
\part $\abs{z-w}\in[4,6]$
\end{parts}
\end{solution}

\newpage

\question[18] 函数~$f(x)=4\sin\dfrac{\pi}{12}x\cdot\sin
    \left(\dfrac{\pi}{2}+\dfrac{\pi}{12}x\right),x\in[a,a+1]$,
    其中常数~$a\in[0,5]$, 求函数~$f(x)$ 的最大值~$g(a)$.

\begin{solution}
略
\end{solution}

\end{questions}
\end{document}